LSAT and Law School Admissions Forum

Get expert LSAT preparation and law school admissions advice from PowerScore Test Preparation.

 Administrator
PowerScore Staff
  • PowerScore Staff
  • Posts: 8916
  • Joined: Feb 02, 2011
|
#32591
Complete Question Explanation

Justify the Conclusion—SN. The correct answer choice is (B)

The stimulus in this question is deceptively wordy, and it conceals a very simple argument. The stimulus begins with the “some people say” rhetorical device, in which the author introduces the theorists’ argument, with which the author disagrees. In this case, the theorists’ position is that “literary critics should strive to be value-neutral in their literary criticism.”

In the second sentence, the author expands on this argument. The theorists believe that literary critics can help readers reach their own conclusions about a literary work if the critics expose the readers to those works without providing them a critical evaluation of the works.

None of this, however, is the stimulus author’s argument. That argument begins in the third sentence, with the premise that “literary criticism cannot be completely value-neutral.” Because of this, the author concludes that “some theorists are mistaken about what is an appropriate goal for literary criticism.”

To fully address this conclusion, we have to refer back to the first sentence, in which the author told us what “some theorists argue.” With that context in mind, the author’s conclusion is that literary critics should not strive to be value-neutral in their literary criticism. All together, the argument is that critics should not strive to be value-neutral in their literary criticism because literary criticism cannot be value-neutral.

As stated at the outset, this is a very simple argument. An apparent weakness in the argument is that the conclusion refers to what “should” or “should not” be done, implying the application of some principle. However, the sole premise did not contain a principle. Rather, it gave only a statement of fact.

The question stem identifies this as a Justify the Conclusion question. Our prephrase is that the correct answer choice will provide a principle that proves the conclusion is valid. To review, the argument contained one premise and one conclusion:
  • Premise: literary criticism cannot be value-neutral
    Conclusion: critics should not strive to be value-neutral in their literary criticism
To prove that the conclusion is valid, with no gray area, the correct answer will say in absolute terms that if literary criticism cannot be value-neutral, then critics should not strive to be value-neutral in their literary criticism. As you may have noticed, our prephrase is expressed as a conditional relationship, which we can diagram as:

LCVN = literary criticism cannot be value-neutral
SVN= critics should not strive to be value-neutral in their literary criticism

..... ..... ..... Sufficient ..... ..... Necessary

..... ..... ..... LCVN ..... :arrow: ..... SVN

Our prephrase is worded in absolute terms because a Justify the Conclusion answer choice is sufficient to prove the conclusion is valid. That level of impact on the conclusion requires a definitively worded answer choice. When, as here, an argument has only one premise offered in support of the conclusion, the author is treating that premise as being sufficient to prove the conclusion is valid. Our prephrase makes explicit that relationship between the sole premise and the conclusion.

Answer choice (A): Recall that the author’s argument actually began with the third sentence in the stimulus, and had nothing to do with helping readers. So, this answer choice is irrelevant to the conclusion.

Answer choice (B): This is the correct answer choice, because it restates the conditional relationship from our prephrase, using synonymous language to produce the same effect.

Answer choice (C): This comparison between works that literary critics like and those they do not like is irrelevant to the conclusion, which had nothing to do with which works literary critics were more likely to critique.

Answer choice (D): Maintaining a proper focus on the distinction between the theorist’ argument and the stimulus author’s shorter argument makes getting rid of this answer choice an easy task. The author’s argument had absolutely nothing to do with the ability of the reader to understand the literary work.

Answer choice (E): As with answer choice (D), the author’s argument had nothing to do with the readers of literary works.
 emilyfoster2013
  • Posts: 9
  • Joined: Oct 22, 2014
|
#17531
As I have been taking more practice tests I am finding myself struggling more with assumption questions than I was before. I am not sure if I am over thinking them, but I am having trouble wrapping my head around the question below:

14. Literary Critics: I chose choice E for this one. I think that the correct answer B seems a little bit too strong. I am not sure where in the stimulus it asserts that if one cannot produce value neutral criticism then one should not even try.

Thank you so much! :)
 Ron Gore
PowerScore Staff
  • PowerScore Staff
  • Posts: 220
  • Joined: May 15, 2013
|
#17542
Hi Emily,

14. Literary Critics: I chose choice E for this one. I think that the correct answer B seems a little bit too strong. I am not sure where in the stimulus it asserts that if one cannot produce value neutral criticism then one should not even try.[/quote]

The key is to recognize that this is a Justify the Conclusion question. By definition, the correct answer choice will be very strong. It will be definitive and remove any doubt that the conclusion is valid.

The only reason the author offered for the conclusion that being value-neutral is not a proper goal for literary criticism is that literary criticism cannot be completely value-neutral. In other words, the author is assuming that "if you can't be completely value-neutral, then you shouldn't try to be value-neutral", which is precisely what answer choice (B) says. Since answer choice (B) contains the rule implicitly applied by the author to reach the conclusion, it proves the conclusion is valid, i.e., "justifies the conclusion".

Does that help?

Ron
 emilyfoster2013
  • Posts: 9
  • Joined: Oct 22, 2014
|
#17597
Yes that helps so much thank you! :)
 jostomel
  • Posts: 12
  • Joined: Jan 09, 2015
|
#18125
Hi,

I am so confused by this question, I know it is supposed to be an easier question, but I am not seeing it at all!! I have no idea why B is correct. During the practice test I completely skipped this question, and now I am having a really hard time even setting this problem up. Could you please help me dissect the stimulus and diagram it out. Thanks
 Lucas Moreau
PowerScore Staff
  • PowerScore Staff
  • Posts: 216
  • Joined: Dec 13, 2012
|
#18134
Hello, jostomel,

The author of this question seems a bit defeatist, doesn't he/she? It's like he/she's saying that since striving for perfection is impossible, striving itself is pointless. But...does that sound right to you? Have you ever heard the phrase "Shoot for the moon; even if you miss, you'll land among the stars"? ;)

It only makes sense if striving for an unreachable goal is meaningless and isn't even worth trying. That's what B is saying. If B is true, then since being value-neutral is impossible, nobody should even try to do it. But if B isn't true, then maybe working towards an unreachable goal might still give you great results. Maybe trying to be value-neutral is itself valuable, even if getting all the way there is impossible. Does that make sense? :-D

Hope that helps,
Lucas Moreau
 jostomel
  • Posts: 12
  • Joined: Jan 09, 2015
|
#18136
Okay, thanks!
 maria34ley
  • Posts: 4
  • Joined: Jan 24, 2018
|
#43655
Can someone please go through the answer choices and explain why they are incorrect and why the correct answer is correct. I am looking for an explanation, as it if was used as an example in the LR Bible for justify conclusion section.

Thank you in advance! :-?
 James Finch
PowerScore Staff
  • PowerScore Staff
  • Posts: 943
  • Joined: Sep 06, 2017
|
#43700
Hi Maria,

This is a difficult question, as the flaw in the stimulus can be hard to puzzle out. At first glance, it looks like valid reasoning, but breaks down when we not the full scope of the conclusion. Here, the conclusion is saying that investigators have not proven that the fire was caused by either campers or lightning, i.e. narrowed down the list of possible causes to only those two. And it reaches that conclusion by stating that the investigators haven't been able to prove that the fire was started by campers, and they haven't been able to prove that the fire was started by lightning.

Can you spot the subtle difference between premise and conclusion? If I don't see a coin flipped, I can't say definitively whether it came up heads or tails, but I can say definitively that it came up either heads or tails. Here we don't know there are only two possible causes for the fire, but we can't rule out that possibility either, nor can we rule out that the two causes that the stimulus tells us have not been proven are the only two possible causes. So our flaw is an evidentiary one: that even though we don't know if two possibilities are correct, they could be the only possibilities, and thus one must be correct. With our flaw identified, we can move on to the answer choices to determine which one matches it:

(A)--Here we are given two possible outcomes, for both of which we lack any evidence indicating they will actually occur. It concludes that there is thus no evidence that either will occur. Thinking through it, this answer choice seems quite similar to the stimulus, in that it also unreasonably concludes that Sada and Brown aren't the only two candidates in the election.

(B)--An attractive wrong answer choice. Similar premises to the stimulus, but a very different and actually valid conclusion. That validity makes this an incorrect answer choice, since there has to be a flaw present in a correct Parallel Flaw answer choice.

(C)--Clear formal logic flaw: "Most" only means a majority, so when we have potentially overlapping groups, both of which contain "most" of the same greater whole, there is a possibility of only a small overlap. For example, if 100 students lived in the dorm, and 51 of them were from out-or-town, and 51 of the same hundred were also engineering majors, only two students would have to be members of both groups. As a flaw completely unrelated to the stimulus, this answer choice is an immediate loser.

(D)--Again, a Formal Logic flaw, one that assumes "some" is less vague than it is, and tries to force two groups into overlapping without sufficient evidence. This answer choice tries to trick test takers by using the same forest/camping setup as the stimulus, but that fluff is irrelevant. Immediate loser.

(E)--Another attractive wrong answer choice. However, it inverts the flawed reasoning in the stimulus, by having the premises give evidence for both events being true, and then concluding that both things could have been true at the same time, when logically (with common outside knowledge) only one should be possible. Two people cannot drive a car at the same time. Regardless, because this doesn't parallel the logic in the stimulus, this answer choice is incorrect.

Hope this clears things up!
 maria34ley
  • Posts: 4
  • Joined: Jan 24, 2018
|
#43759
Hi James,

Again, very thorough explanation. Thank you!!

Maria

Get the most out of your LSAT Prep Plus subscription.

Analyze and track your performance with our Testing and Analytics Package.